To evaluate the limit.











up vote
-1
down vote

favorite
1













Evaluate
$$lim_{ntoinfty}left[left(frac1nright)^n+left(frac2nright)^n+cdots+left(frac{n-1}nright)^n+left(frac nnright)^nright]$$




I tried to solve this by taking logs on both sides and got this form:
$$log l = lim_{ntoinfty} nlogfrac{n!}{n^n}=lim_{ntoinfty}nlogfrac{n!}{n^n}$$
After applying limit, $frac{n!}{n^n}to0$, so $logfrac{n!}{n^n}to-infty$ as $ntoinfty$. Then I am stuck. How to solve these type of problems?










share|cite|improve this question
























  • Better formatting please. Also, how did taking log of a sum convert it into a product?
    – Aditya Dua
    Nov 18 at 3:10










  • Yes, I made mistake.
    – Mathsaddict
    Nov 18 at 3:16










  • @Mathsaddict: I just edit your question for a good look. Is I'm missing anything?
    – Chinnapparaj R
    Nov 18 at 3:19










  • @Chinnapparaj you didn't miss anything. It looks better now. Thanks for edit.
    – Mathsaddict
    Nov 18 at 3:22






  • 1




    It is $lim_{n→∞}frac{1}{n^2}.frac{n(n+1)(2n+1)}{6}=∞$
    – sirous
    Nov 18 at 3:56















up vote
-1
down vote

favorite
1













Evaluate
$$lim_{ntoinfty}left[left(frac1nright)^n+left(frac2nright)^n+cdots+left(frac{n-1}nright)^n+left(frac nnright)^nright]$$




I tried to solve this by taking logs on both sides and got this form:
$$log l = lim_{ntoinfty} nlogfrac{n!}{n^n}=lim_{ntoinfty}nlogfrac{n!}{n^n}$$
After applying limit, $frac{n!}{n^n}to0$, so $logfrac{n!}{n^n}to-infty$ as $ntoinfty$. Then I am stuck. How to solve these type of problems?










share|cite|improve this question
























  • Better formatting please. Also, how did taking log of a sum convert it into a product?
    – Aditya Dua
    Nov 18 at 3:10










  • Yes, I made mistake.
    – Mathsaddict
    Nov 18 at 3:16










  • @Mathsaddict: I just edit your question for a good look. Is I'm missing anything?
    – Chinnapparaj R
    Nov 18 at 3:19










  • @Chinnapparaj you didn't miss anything. It looks better now. Thanks for edit.
    – Mathsaddict
    Nov 18 at 3:22






  • 1




    It is $lim_{n→∞}frac{1}{n^2}.frac{n(n+1)(2n+1)}{6}=∞$
    – sirous
    Nov 18 at 3:56













up vote
-1
down vote

favorite
1









up vote
-1
down vote

favorite
1






1






Evaluate
$$lim_{ntoinfty}left[left(frac1nright)^n+left(frac2nright)^n+cdots+left(frac{n-1}nright)^n+left(frac nnright)^nright]$$




I tried to solve this by taking logs on both sides and got this form:
$$log l = lim_{ntoinfty} nlogfrac{n!}{n^n}=lim_{ntoinfty}nlogfrac{n!}{n^n}$$
After applying limit, $frac{n!}{n^n}to0$, so $logfrac{n!}{n^n}to-infty$ as $ntoinfty$. Then I am stuck. How to solve these type of problems?










share|cite|improve this question
















Evaluate
$$lim_{ntoinfty}left[left(frac1nright)^n+left(frac2nright)^n+cdots+left(frac{n-1}nright)^n+left(frac nnright)^nright]$$




I tried to solve this by taking logs on both sides and got this form:
$$log l = lim_{ntoinfty} nlogfrac{n!}{n^n}=lim_{ntoinfty}nlogfrac{n!}{n^n}$$
After applying limit, $frac{n!}{n^n}to0$, so $logfrac{n!}{n^n}to-infty$ as $ntoinfty$. Then I am stuck. How to solve these type of problems?







limits






share|cite|improve this question















share|cite|improve this question













share|cite|improve this question




share|cite|improve this question








edited Nov 18 at 4:34









Rócherz

2,6612721




2,6612721










asked Nov 18 at 3:03









Mathsaddict

607




607












  • Better formatting please. Also, how did taking log of a sum convert it into a product?
    – Aditya Dua
    Nov 18 at 3:10










  • Yes, I made mistake.
    – Mathsaddict
    Nov 18 at 3:16










  • @Mathsaddict: I just edit your question for a good look. Is I'm missing anything?
    – Chinnapparaj R
    Nov 18 at 3:19










  • @Chinnapparaj you didn't miss anything. It looks better now. Thanks for edit.
    – Mathsaddict
    Nov 18 at 3:22






  • 1




    It is $lim_{n→∞}frac{1}{n^2}.frac{n(n+1)(2n+1)}{6}=∞$
    – sirous
    Nov 18 at 3:56


















  • Better formatting please. Also, how did taking log of a sum convert it into a product?
    – Aditya Dua
    Nov 18 at 3:10










  • Yes, I made mistake.
    – Mathsaddict
    Nov 18 at 3:16










  • @Mathsaddict: I just edit your question for a good look. Is I'm missing anything?
    – Chinnapparaj R
    Nov 18 at 3:19










  • @Chinnapparaj you didn't miss anything. It looks better now. Thanks for edit.
    – Mathsaddict
    Nov 18 at 3:22






  • 1




    It is $lim_{n→∞}frac{1}{n^2}.frac{n(n+1)(2n+1)}{6}=∞$
    – sirous
    Nov 18 at 3:56
















Better formatting please. Also, how did taking log of a sum convert it into a product?
– Aditya Dua
Nov 18 at 3:10




Better formatting please. Also, how did taking log of a sum convert it into a product?
– Aditya Dua
Nov 18 at 3:10












Yes, I made mistake.
– Mathsaddict
Nov 18 at 3:16




Yes, I made mistake.
– Mathsaddict
Nov 18 at 3:16












@Mathsaddict: I just edit your question for a good look. Is I'm missing anything?
– Chinnapparaj R
Nov 18 at 3:19




@Mathsaddict: I just edit your question for a good look. Is I'm missing anything?
– Chinnapparaj R
Nov 18 at 3:19












@Chinnapparaj you didn't miss anything. It looks better now. Thanks for edit.
– Mathsaddict
Nov 18 at 3:22




@Chinnapparaj you didn't miss anything. It looks better now. Thanks for edit.
– Mathsaddict
Nov 18 at 3:22




1




1




It is $lim_{n→∞}frac{1}{n^2}.frac{n(n+1)(2n+1)}{6}=∞$
– sirous
Nov 18 at 3:56




It is $lim_{n→∞}frac{1}{n^2}.frac{n(n+1)(2n+1)}{6}=∞$
– sirous
Nov 18 at 3:56










1 Answer
1






active

oldest

votes

















up vote
4
down vote



accepted










Correct version of the question:
$$lim_{ntoinfty}sum_{k=1}^{n}left(frac{k}{n}right)^n=? $$
And this sum converges to
$$
lim_{ntoinfty}sum_{k=0}^{infty}left(frac{n-k}{n}right)^n1_{kleq n} = sum_{k=0}^infty lim_{ntoinfty}left(frac{n-k}{n}right)^n1_{kleq n} = sum_{k=0}^infty e^{-k} = frac{1}{1-e^{-1}},
$$
by (Lebesgue's) dominated convergence theorem or monotone convergence theorem. (Note: $(1-frac{k}{n})^n uparrow e^{-k}$ as $nto infty$.)






share|cite|improve this answer























  • I didn't understand the transformation from summation(k=1 to n) (k/n)^n to summation(k=0 to ∞) ((n-k)/n)^n
    – Mathsaddict
    Nov 18 at 4:53










  • Sorry that I didn't explain the notation $1_{kleq n}.$ This 'indicator' function takes value $1$ if $kleq n$ and $0$ otherwise, so infinite sum is actually calculated only for $0leq kleq n$.
    – Song
    Nov 18 at 4:57












  • Okay, I got it. Thanks.
    – Mathsaddict
    Nov 18 at 5:04






  • 1




    There can be possibly so many different ways to get the limit result of the given sum, but one of the things that you should do first is to investigate the behavior of each term, guess how the total sum would behave and choose/find the appropriate tools/theorems that can show your intuition rigorously.
    – Song
    Nov 18 at 9:30











Your Answer





StackExchange.ifUsing("editor", function () {
return StackExchange.using("mathjaxEditing", function () {
StackExchange.MarkdownEditor.creationCallbacks.add(function (editor, postfix) {
StackExchange.mathjaxEditing.prepareWmdForMathJax(editor, postfix, [["$", "$"], ["\\(","\\)"]]);
});
});
}, "mathjax-editing");

StackExchange.ready(function() {
var channelOptions = {
tags: "".split(" "),
id: "69"
};
initTagRenderer("".split(" "), "".split(" "), channelOptions);

StackExchange.using("externalEditor", function() {
// Have to fire editor after snippets, if snippets enabled
if (StackExchange.settings.snippets.snippetsEnabled) {
StackExchange.using("snippets", function() {
createEditor();
});
}
else {
createEditor();
}
});

function createEditor() {
StackExchange.prepareEditor({
heartbeatType: 'answer',
convertImagesToLinks: true,
noModals: true,
showLowRepImageUploadWarning: true,
reputationToPostImages: 10,
bindNavPrevention: true,
postfix: "",
imageUploader: {
brandingHtml: "Powered by u003ca class="icon-imgur-white" href="https://imgur.com/"u003eu003c/au003e",
contentPolicyHtml: "User contributions licensed under u003ca href="https://creativecommons.org/licenses/by-sa/3.0/"u003ecc by-sa 3.0 with attribution requiredu003c/au003e u003ca href="https://stackoverflow.com/legal/content-policy"u003e(content policy)u003c/au003e",
allowUrls: true
},
noCode: true, onDemand: true,
discardSelector: ".discard-answer"
,immediatelyShowMarkdownHelp:true
});


}
});














draft saved

draft discarded


















StackExchange.ready(
function () {
StackExchange.openid.initPostLogin('.new-post-login', 'https%3a%2f%2fmath.stackexchange.com%2fquestions%2f3003092%2fto-evaluate-the-limit%23new-answer', 'question_page');
}
);

Post as a guest















Required, but never shown

























1 Answer
1






active

oldest

votes








1 Answer
1






active

oldest

votes









active

oldest

votes






active

oldest

votes








up vote
4
down vote



accepted










Correct version of the question:
$$lim_{ntoinfty}sum_{k=1}^{n}left(frac{k}{n}right)^n=? $$
And this sum converges to
$$
lim_{ntoinfty}sum_{k=0}^{infty}left(frac{n-k}{n}right)^n1_{kleq n} = sum_{k=0}^infty lim_{ntoinfty}left(frac{n-k}{n}right)^n1_{kleq n} = sum_{k=0}^infty e^{-k} = frac{1}{1-e^{-1}},
$$
by (Lebesgue's) dominated convergence theorem or monotone convergence theorem. (Note: $(1-frac{k}{n})^n uparrow e^{-k}$ as $nto infty$.)






share|cite|improve this answer























  • I didn't understand the transformation from summation(k=1 to n) (k/n)^n to summation(k=0 to ∞) ((n-k)/n)^n
    – Mathsaddict
    Nov 18 at 4:53










  • Sorry that I didn't explain the notation $1_{kleq n}.$ This 'indicator' function takes value $1$ if $kleq n$ and $0$ otherwise, so infinite sum is actually calculated only for $0leq kleq n$.
    – Song
    Nov 18 at 4:57












  • Okay, I got it. Thanks.
    – Mathsaddict
    Nov 18 at 5:04






  • 1




    There can be possibly so many different ways to get the limit result of the given sum, but one of the things that you should do first is to investigate the behavior of each term, guess how the total sum would behave and choose/find the appropriate tools/theorems that can show your intuition rigorously.
    – Song
    Nov 18 at 9:30















up vote
4
down vote



accepted










Correct version of the question:
$$lim_{ntoinfty}sum_{k=1}^{n}left(frac{k}{n}right)^n=? $$
And this sum converges to
$$
lim_{ntoinfty}sum_{k=0}^{infty}left(frac{n-k}{n}right)^n1_{kleq n} = sum_{k=0}^infty lim_{ntoinfty}left(frac{n-k}{n}right)^n1_{kleq n} = sum_{k=0}^infty e^{-k} = frac{1}{1-e^{-1}},
$$
by (Lebesgue's) dominated convergence theorem or monotone convergence theorem. (Note: $(1-frac{k}{n})^n uparrow e^{-k}$ as $nto infty$.)






share|cite|improve this answer























  • I didn't understand the transformation from summation(k=1 to n) (k/n)^n to summation(k=0 to ∞) ((n-k)/n)^n
    – Mathsaddict
    Nov 18 at 4:53










  • Sorry that I didn't explain the notation $1_{kleq n}.$ This 'indicator' function takes value $1$ if $kleq n$ and $0$ otherwise, so infinite sum is actually calculated only for $0leq kleq n$.
    – Song
    Nov 18 at 4:57












  • Okay, I got it. Thanks.
    – Mathsaddict
    Nov 18 at 5:04






  • 1




    There can be possibly so many different ways to get the limit result of the given sum, but one of the things that you should do first is to investigate the behavior of each term, guess how the total sum would behave and choose/find the appropriate tools/theorems that can show your intuition rigorously.
    – Song
    Nov 18 at 9:30













up vote
4
down vote



accepted







up vote
4
down vote



accepted






Correct version of the question:
$$lim_{ntoinfty}sum_{k=1}^{n}left(frac{k}{n}right)^n=? $$
And this sum converges to
$$
lim_{ntoinfty}sum_{k=0}^{infty}left(frac{n-k}{n}right)^n1_{kleq n} = sum_{k=0}^infty lim_{ntoinfty}left(frac{n-k}{n}right)^n1_{kleq n} = sum_{k=0}^infty e^{-k} = frac{1}{1-e^{-1}},
$$
by (Lebesgue's) dominated convergence theorem or monotone convergence theorem. (Note: $(1-frac{k}{n})^n uparrow e^{-k}$ as $nto infty$.)






share|cite|improve this answer














Correct version of the question:
$$lim_{ntoinfty}sum_{k=1}^{n}left(frac{k}{n}right)^n=? $$
And this sum converges to
$$
lim_{ntoinfty}sum_{k=0}^{infty}left(frac{n-k}{n}right)^n1_{kleq n} = sum_{k=0}^infty lim_{ntoinfty}left(frac{n-k}{n}right)^n1_{kleq n} = sum_{k=0}^infty e^{-k} = frac{1}{1-e^{-1}},
$$
by (Lebesgue's) dominated convergence theorem or monotone convergence theorem. (Note: $(1-frac{k}{n})^n uparrow e^{-k}$ as $nto infty$.)







share|cite|improve this answer














share|cite|improve this answer



share|cite|improve this answer








edited Nov 18 at 4:44

























answered Nov 18 at 4:36









Song

82011




82011












  • I didn't understand the transformation from summation(k=1 to n) (k/n)^n to summation(k=0 to ∞) ((n-k)/n)^n
    – Mathsaddict
    Nov 18 at 4:53










  • Sorry that I didn't explain the notation $1_{kleq n}.$ This 'indicator' function takes value $1$ if $kleq n$ and $0$ otherwise, so infinite sum is actually calculated only for $0leq kleq n$.
    – Song
    Nov 18 at 4:57












  • Okay, I got it. Thanks.
    – Mathsaddict
    Nov 18 at 5:04






  • 1




    There can be possibly so many different ways to get the limit result of the given sum, but one of the things that you should do first is to investigate the behavior of each term, guess how the total sum would behave and choose/find the appropriate tools/theorems that can show your intuition rigorously.
    – Song
    Nov 18 at 9:30


















  • I didn't understand the transformation from summation(k=1 to n) (k/n)^n to summation(k=0 to ∞) ((n-k)/n)^n
    – Mathsaddict
    Nov 18 at 4:53










  • Sorry that I didn't explain the notation $1_{kleq n}.$ This 'indicator' function takes value $1$ if $kleq n$ and $0$ otherwise, so infinite sum is actually calculated only for $0leq kleq n$.
    – Song
    Nov 18 at 4:57












  • Okay, I got it. Thanks.
    – Mathsaddict
    Nov 18 at 5:04






  • 1




    There can be possibly so many different ways to get the limit result of the given sum, but one of the things that you should do first is to investigate the behavior of each term, guess how the total sum would behave and choose/find the appropriate tools/theorems that can show your intuition rigorously.
    – Song
    Nov 18 at 9:30
















I didn't understand the transformation from summation(k=1 to n) (k/n)^n to summation(k=0 to ∞) ((n-k)/n)^n
– Mathsaddict
Nov 18 at 4:53




I didn't understand the transformation from summation(k=1 to n) (k/n)^n to summation(k=0 to ∞) ((n-k)/n)^n
– Mathsaddict
Nov 18 at 4:53












Sorry that I didn't explain the notation $1_{kleq n}.$ This 'indicator' function takes value $1$ if $kleq n$ and $0$ otherwise, so infinite sum is actually calculated only for $0leq kleq n$.
– Song
Nov 18 at 4:57






Sorry that I didn't explain the notation $1_{kleq n}.$ This 'indicator' function takes value $1$ if $kleq n$ and $0$ otherwise, so infinite sum is actually calculated only for $0leq kleq n$.
– Song
Nov 18 at 4:57














Okay, I got it. Thanks.
– Mathsaddict
Nov 18 at 5:04




Okay, I got it. Thanks.
– Mathsaddict
Nov 18 at 5:04




1




1




There can be possibly so many different ways to get the limit result of the given sum, but one of the things that you should do first is to investigate the behavior of each term, guess how the total sum would behave and choose/find the appropriate tools/theorems that can show your intuition rigorously.
– Song
Nov 18 at 9:30




There can be possibly so many different ways to get the limit result of the given sum, but one of the things that you should do first is to investigate the behavior of each term, guess how the total sum would behave and choose/find the appropriate tools/theorems that can show your intuition rigorously.
– Song
Nov 18 at 9:30


















draft saved

draft discarded




















































Thanks for contributing an answer to Mathematics Stack Exchange!


  • Please be sure to answer the question. Provide details and share your research!

But avoid



  • Asking for help, clarification, or responding to other answers.

  • Making statements based on opinion; back them up with references or personal experience.


Use MathJax to format equations. MathJax reference.


To learn more, see our tips on writing great answers.





Some of your past answers have not been well-received, and you're in danger of being blocked from answering.


Please pay close attention to the following guidance:


  • Please be sure to answer the question. Provide details and share your research!

But avoid



  • Asking for help, clarification, or responding to other answers.

  • Making statements based on opinion; back them up with references or personal experience.


To learn more, see our tips on writing great answers.




draft saved


draft discarded














StackExchange.ready(
function () {
StackExchange.openid.initPostLogin('.new-post-login', 'https%3a%2f%2fmath.stackexchange.com%2fquestions%2f3003092%2fto-evaluate-the-limit%23new-answer', 'question_page');
}
);

Post as a guest















Required, but never shown





















































Required, but never shown














Required, but never shown












Required, but never shown







Required, but never shown

































Required, but never shown














Required, but never shown












Required, but never shown







Required, but never shown







Popular posts from this blog

QoS: MAC-Priority for clients behind a repeater

Ивакино (Тотемский район)

Can't locate Autom4te/ChannelDefs.pm in @INC (when it definitely is there)